76
7 th /8 th Review Project Collection

7 th /8 th Review Project Collection

Embed Size (px)

DESCRIPTION

7 th /8 th Review Project Collection. LO 1.29. The student is able to describe the reasons for revisions of scientific hypotheses of the origin of life on Earth. SP 6.3. The student can articulate the reasons that scientific explanations and theories are refined or replaced. - PowerPoint PPT Presentation

Citation preview

Page 1: 7 th /8 th  Review Project Collection

7th/8th Review Project Collection

Page 2: 7 th /8 th  Review Project Collection

• LO 1.29. The student is able to describe the reasons for revisions of scientific hypotheses of the origin of life on Earth. • SP 6.3. The student can articulate the reasons that scientific explanations and theories are• refined or replaced.

• Explanation. Science is constantly changing. We are continually gaining knowledge about life on Earth, partly due to the development of new technologies. Experimental investigations allow us to learn about the origin of life on earth. With the help of previous learned knowledge and current technologies, more details and information can be gathered about life on Earth and how life came to be. Conclusions can be drawn from looking at historical evidence and past experiments, as well as current findings and explorations. Because more and more information is found by scientific research everyday, revisions to hypotheses are often made because previous hypotheses are proven to be incorrect or in need of “revisions.”In scientific research, making a hypothesis is an important step before performing an experiment or conducting research because it addresses the purpose of the experiment and helps the student to apply current knowledge and the historical background of the claim with future expectations.

• • MC Question• Hypotheses are made before every science exploration or experiment. As time progresses and science advances, more knowledge about the origin of life on Earth

becomes known. With more knowledge available, scientific hypotheses are revised over time because some information is proven false, inconclusive, or just not as detailed as it could be. There are always new improvements with science. Listed below are 4 scientific hypotheses about the origin of life on earth. Which of the following represents a possible hypothesis of the origin of life using up-to-date and current knowledge

• A)Life on Earth originated at photosynthetically-active porous structures, similar to deep-sea hydrothermal vents, made of zinc sulfide (more commonly known as phosphor).

• B)Prions were a first model for life on earth. • C)The early Earth had a reducing atmosphere, which meant it had large amounts of hydrogen and almost no oxygen.• D)RNA alone triggered the rise of life from a sea of molecules.• E)Parasites are the model for ‘first life’

• Free Response Question• Suppose you live in the era where Earth’s atmosphere was thought to be made up of just oxygen. Describe how you would revise this hypothesis with knowledge of

the 21st Century. What makes this hypothesis so outdated? Support with an example of an important gas found in the atmosphere and it’s purpose in life. Why are theories and hypotheses constantly changing in our world?

Referenceshttp://undsci.berkeley.edu/article/0_0_0/howscienceworks_06

http://echo2.epfl.ch/VICAIRE/mod_2/chapt_3/main.htm

http://www.cawcr.gov.au/projects/climatechange/challengesCoasts.shtml

Page 3: 7 th /8 th  Review Project Collection

MC Question Answer/ExplanationA)It is argued that under the high pressure of a carbon-dioxide-dominated atmosphere, zinc sulfide structures could form on the surface of the first continents, where they had access to sunlight. This is an updated hypothesis. You can tell because new knowledge about certain chemicals and gases is incorporated and applied to the origin of life hypothesis. B) Prions were not one of the first models on earth because they can only ‘replicate’ by causing proteins made by a cell to become misshapedC) An oxygen-poor atmosphere would have resulted in an atmosphere full of carbon monoxide, noxious methane, hydrogen sulfide, and ammonia, making life on earth difficult and ‘deadly’.D)RNA is a very complex molecule that is unlikely to have arisen prebiotically. Also, RNA is also considered inherently unstable.E)A parasite can not be one of the first life models because in order for a parasite to survive, it needs another cell.

Free Response AnswerSuppose you live in the era where Earth’s atmosphere was thought to be made up of just oxygen. Describe how you would revise this hypothesis with knowledge of the 21st Century. What makes this hypothesis so outdated? Support with an example of an important gas found in the atmosphere and it’s purpose in life. Why are theories and hypotheses constantly changing in our world?

The earth’s atmosphere is now known to have many key components. The atmosphere is made up of 79% Nitrogen, 20% oxygen and 1% of other gases. If there was no nitrogen(also considered a buffer gas), then the atmosphere would be made up of only oxygen and that would cause combustible material on earth to explode from even a tiny spark. Living organisms also require nitrogen, which is an essential component of proteins and nucleic acids, for example. This hypothesis must be revised to say that the earth’s atmosphere is made up of more than just oxygen, but also other key elements necessary for life, such as nitrogen. The old hypothesis is outdated because the information is incorrect and relatively vague. Today, we have so much more knowledge of different elements and gases and how they interact in the circle of life. Theories and hypotheses are constantly changing because new information and knowledge is discovered every day. Some new information proves previous hypotheses/theories incorrect, other new information may not be entirely incorrect, but more developed from improvements in technology and have greater details and proof.

Page 4: 7 th /8 th  Review Project Collection

LO 3.48: The student is able to create a visual representation to describe how nervous systems detect external and internal signals.

SP 1.1: The student can create representation and models of natural or man-made phenomena and systems in the domain.

Explanation: The neuron is the basic unit in the nervous system. A labeled model of a neuron is shown in an image below. The dendrites bring the signal, whether it’s external or internal, toward the cell body. From the cell body, the signal travels down the axon while the myelin sheath insulates the axon. This is done through an action potential when the sodium channels allow enough sodium into the neuron where it becomes -55mV and a nerve impulse is fired. The neuron must re-polarize itself by opening the potassium channels to regulate the cell back to -70mV. The cell remains at a constant -70mV with the help of the Na+/K+ pumps that use ATP to make sure the concentration is the same on both sides of the membrane. The impulse then travels down the axon until it reaches the synapse. The voltage-gated ion channels open to allow calcium to make the neurotransmitters cross the synaptic cleft and bind to the receptor on the postsynaptic cell. It will allow sodium to come in or potassium to leave and then the channels will close.

M.C. Question: Which of the following happens when the action potential reaches the terminus of the pre-synaptic cell?

A) The ligand-gated ion channels open B) Allow potassium to leave the cellC) It depolarizes the cellD) The voltage-gated calcium channels open

Learning Log/ FRQ-Style Question: Explain what happens when the doctor taps your knee and causes your lower leg to jerk and start swinging. (CUES: reflexes, quads, sensory neurons, brain, spinal chord, motor neurons)

Page 5: 7 th /8 th  Review Project Collection

Answer Key- LO 3.48M.C. Question: Which of the following happens when the action potential reaches the terminus

of the pre-synaptic cell?A) The ligand-gated ion channels open B) Allow potassium to leave the cellC) It depolarizes the cellD) The voltage-gated calcium channels open

Explain what happens when the doctor taps your knee and causes your lower leg to jerk and start swinging. (CUES: reflexes, quads, sensory neurons, brain, spinal chord, motor neurons)

When the doctors hits your knee, sensors detect a stretch in your quad muscle which makes sensory neurons send a signal to your spinal chord. The signal is split to influence the motor neurons. Therefore when the sensory neurons contracts the quad the motor neurons make the hamstring relax. Even though there are many different processes going on the signal never ends up reaching the brain. This is why there is a reflex when your doctor hits you in the correct place on the knee.

Page 6: 7 th /8 th  Review Project Collection

Learning Objective 4.7: The student is able to refine representations to illustrate how interactions between external stimuli and gene expression result in specialization of cells, tissues, and organs. Science Practice 1.3: The student can refine representations and models of natural or man-made phenomena and systems in the domain. Statement: Cells specialize their function and structure through the stimulation and repression of coding DNA sequences by regulatory proteins. Regulatory proteins are coded from regulatory genes, any sequence of DNA that codes for regulator proteins or RNA. Regulatory proteins can be activated by external stimuli such as temperature or nutrient levels in the cytoplasm, increasing or decreasing the rate of prevalence for a protein that is transcribed by a particular gene sequence as the genes has other regulatory proteins bound or unbound to it in response to the stimuli. During the process of specialization, the external environment of a cell has an important effect on the development on that cell. Ligands produced by other cells bind with membrane bound receptors to activate signal transduction pathways and cause cellular responses through induction. These cellular reaction generating molecules also exist in the cell that is being affected, with their presence creating a cycle of development as the proteins their genes code for elicit more responses in conjunction with external molecules in order to specialize the cell. Specialization is the changing of function and structure of a cell for the greatest efficiency in a performing a particular task in conjunction with other cells to form a specific type of tissue. As an example of the effect of environmental stimuli on specialization, take into account a pair of identical twins. Since the twins are identical, they share the same genes. If, however, one of the twins was several inches taller with noticeably darker skin tone and as such exhibited a difference in phenotype despite having the same genes. Both of these traits would be caused by an environmental stimulus or condition that affected gene expression rather than solely being affected by the cell’s internal environment. For specific cellular specialization, neurons could be taken for an example. Neurons are structurally adapted for sending electrical impulses and for communication, being elongated with numerous dendrites and axon terminals for multiple connections to other neurons. Their interior chemical environment is different, specialized for sodium and potassium flow for action potentials to be sent.

Other factors

http://www.biologyjunction.com/images/whatis.jpg

Page 7: 7 th /8 th  Review Project Collection

Review Questions

Induction is the process of cells sending signals into their environment to affect the development of nearby cells through signal transduction pathways. What is true about this process?A.Signals produced by neighboring cells suppress transcription of regulatory genes during development.B.Signals produced by neighboring cells stimulate transcription of regulatory genes during development.C.Signals produced by neighboring cells either promote or suppress transcription of regulatory genes during development.D.Signals produced by neighboring cells create positive feedback loops to influence their own development.

This muscular tissue, a collection of muscle cells, does not resemble the typical image of a cell presented by the media. Name two advantages muscle cells have by having this structure. In addition, name an internal adaptation that muscle cells to more effectively perform their task.

http://medicalpicturesinfo.com/muscle-cell/

Page 8: 7 th /8 th  Review Project Collection

Answer key

Induction is the process of cells sending signals into their environment to affect the development of nearby cells through signal transduction pathways. What is true about this process?A. Signals produced by neighboring cells suppress transcription of regulatory genes during development.B. Signals produced by neighboring cells stimulate transcription of regulatory genes during development.C. Signals produced by neighboring cells either promote or suppress transcription of regulatory genes during development.D. Signals produced by neighboring cells create positive feedback loops to influence their own development

This muscular tissue, a collection of muscle cells, does not resemble the typical image of a cell presented by the media. Describe two advantages muscle cells have by having this structure. In addition, name an internal adaptation that muscle cells to more effectively perform their task and describe how it improves function.

The elongated design of muscle cells allows for the extended contraction and extension of muscles, and are lengthy to reach across the extent of bones so that they are able to pull them.

Muscle fibers are thick and are able to endure more stress than other cells and are comparatively more difficult to damage because they have to regularly move.

Muscle cells have increased concentrations of mitochondria within their cytoplasm to compensate for their high levels of energy use, they must produce ATP in larger amounts in order to function.

Page 9: 7 th /8 th  Review Project Collection

LO 3.11 The student is able to evaluate evidence provided by data sets to support the claim that heritable information is passes from one generation to another generation through mitosis, or meiosis followed by fertilization. SP 5.3 The student can evaluate the evidence provided by data sets in relation to a particular scientific question. Explanation:The transmission of genetic information is passed from one generation to the next through sexual reproduction. Genetic variation is the result of sexual reproduction because new combinations of genetic information are created, due to the activity of chromosomes during meiosis. Genetic material is divided during nuclear division either through the process of mitosis or meiosis. Meiosis, a reduction division, produces haploid gametes that are each genetically different from the parent cell and each other. These haploid cells produced are gametes. When the gametes fuse together fertilization occurs. This then gives rise to a diploid cell known as the zygote. The zygote will then divide by mitosis in order to produce a multicellular organism. Multiple Choice Question:Which of the following statement (s) are false concerning the inheritance of genetic information via mitosis?A) The daughter cell chromosomes are identical to the parent cell B) The daughter cells of mitosis are haploidC) The daughter cells of mitosis are diploidD) Variation of inherited material is introduced via crossing in mitosis

Learning Log Question:

Normal human somatic cells contain 46 chromosomes. One set of 23 is derived from the mother, while the other set is from the father.

A) After a cell undergoes mitosis, how many chromosomes will the cell contain?

B) After meiosis takes place, how many cells will be produced and how many chromosomes will they contain?

C) How is genetic variation attained through sexual reproduction via fertilization?

Page 10: 7 th /8 th  Review Project Collection

Answer Key- LO 3.11Which of the following statement (s) are false concerning the inheritance of

genetic information via mitosis?A) The daughter cell chromosomes are identical to the parent cell B) The daughter cells of mitosis are haploidC) The daughter cells of mitosis are diploidD) Variation of inherited material is introduced via crossing in mitosis

Learning Log Question: Normal human somatic cells contain 46 chromosomes. One set of 23 is derived

from the mother, while the other set is from the father. A) After a cell undergoes mitosis, how many chromosomes will the cell contain?B) After meiosis takes place, how many cells will be produced and how many

chromosomes will they contain?C) How is genetic variation attained through sexual reproduction via

fertilization?Answers:A) The cell will contain 46 chromosomes because two genetically identical

cells are produced.B) Four daughter cells are produced, containing half as many chromosomes as

the parent cell. Therefore, 23 chromosomes are in these gametes. C) During meiosis crossing over can occur. Crossing over is when

homologous chromatids exchange genetic information. Also, the random nature of fertilization only increases the genetic variation. For an example, each female and male gamete represents one out of approximately 8 million chromosome combinations.

Page 11: 7 th /8 th  Review Project Collection

SP:1.1 The student can create representations and models of natural or man-made phenomena and systems in the domain.

LO:4.1: The student is able to create a representation that describes how organisms exchange information in response to internal changes and external cues, and which can result in changes in behavior.

Explanation: For example, plants communicate and exchange information through the release of chemicals. The roots exchange gases with the air spaces of soil, then take in oxygen and discharging CO2, while cellular respiration breaks down sugars. Information in response to internal changes and external cues are detected by receptors at first. If the plant does not receive enough light, or photosynthesis, the plant will respond negatively. Without light or water the plant would slowly shrivel up, but if the shoot reaches sunlight, it will begin to green, or de-etiolation. This relationship is extremely important for the plants survival.M.C. Question: Which of the following statements concerning Photosystem II is false?A) During the electron transport chain 2NADP+ to 2NADPH is not reduced.B) Uses light energy to oxidize two molecules of water into one molecule of molecular oxygen. C) Absorbs photons of a wavelength of 680 nmD) During the electron transport process a proton gradient is generated across the thylakoid membrane.

Learning Log/FRQ-style Question: List 2 examples of stimuli that plants react to and then describe the process that occurs in great detail.

Page 12: 7 th /8 th  Review Project Collection

Answer Key- LO 3.41Which of the following statements concerning Photosystem II is false?

A) During the electron transport chain 2NADP+ to 2NADPH is not reduced.B) Uses light energy to oxidize two molecules of water into one molecule of molecular oxygen. C) Absorbs photons of a wavelength of 680 nmD) During the electron transport process a proton gradient is generated across the thylakoid membraneExplanation: The correct answer is A because the 4 electrons removed from the water molecules are transferred by an electron transport chain to reduce 2NADP+ to 2NADPH.

List 2 examples of stimuli that plants react to and then describe the process that occurs in great detail.

One example of a stimuli that plants react to is Hydrotropism, which is the plant’s response to water. When hydrotropism occurs the roots can grow towards water (positive) or the shoots can grow away from water (negative). Auxins, a plant hormone is what correlates this growth. They play a huge role is the bending of the plants.

Another example is Phototropism, which is a plants response to light. Light is very important for the survival of plants, and photosynthesis take place in the chloroplast of plants. The chemical equation for photosynthesis is CO2 + H2O + light energy ------> C6H12O6 + O2 + energy. Once photosynthesis takes place, carbon dioxide is converted into organic compounds, which are then used for the reproduction and growth of plants.

http://www.plant-and-flower-guide.com/phototropism.html

Page 13: 7 th /8 th  Review Project Collection

LO 2.34: The student is able to describe the role of programmed cell death in development and differentiation, the reuse of molecules, and the maintenance of dynamic homeostasis.

SP 7.1: The student can connect phenomena and models across spatial and temporal scales

Explanation: Apoptosis is the body sending a signal to a cell, causing the cell to die. The formation of digits is determined by this programmed cell death of the interdigital regions between the digits. Separation of the cartilaginous condensations which are to become digits depend on the death of the mesenchymal cells that are in between these cells . This allows the adjacent cells to differentiate into the specific , individual digit or finger on the hand rather than be connected by webbing. This is why early developing fetuses have webbing between their fingers. When the cell undergoes apoptosis a specific enzyme will break down the proteins, organelles, and DNA. These parts are digested by vesicles and reused as building blocks for new cells. Apoptosis plays a vital role in the maintaining homeostasis as if it occurs too much or too little can lead to cancer or disease. For example if the DNA of a cell is damaged and cannot be repaired the cell must perform apoptosis or risk the defect being replicated and spread to other cells.

MC Question: Which of the following will trigger a cell major caspase activation pathway causing apoptosis?A-cAMPB- cytocinesC- cytochrome CD- apoptase

FR Question: What medical problems could arise from the disruption of programmed cell death? Explain why these problems would occur?

Page 14: 7 th /8 th  Review Project Collection

Answer Key- LO 2.34

MC Question: Which of the following will trigger a cell major caspase activation pathway causing apoptosis?A-cAMPB- cytokinesC- cytochrome CD- apoptase

What medical problems could arise from the disruption of programmed cell death? Explain why these problems would occur?

Some problems associated with disruption in programmed cell death are cancer, ALS, and huntington’s disease. Cancer can occur if programmed cell death does not occur as frequently as it should. This is because a cell, which has a defect of some sort that should undergo apoptosis, does not and passes the defect as it replicates leading to a collection of defective tissues or cancer. ALS and Huntington’s disease can occur from programmed cell death occuring more frequently than it should. Healthy cells are given the signal for apoptosis and degrade themselves. This can destroy healthy neurological pathways leading to a loss of functionality in an individual.

Page 15: 7 th /8 th  Review Project Collection

LO 4.25: The student is able to use evidence to justify a claim that a variety of phenotypic responses to a single environmental factor can result from different genotypes within the population.

SP 6.1: The student can justify claims with evidence.Explanation: A populations ability to respond to one environmental factor differently is occurring because of genetic variation within the

population. Populations that are not able to have genetic variations are in danger of becoming extinct. Some examples of populations that didn’t have genetic diversity are the California Condor, the Black-footed Ferret, and Prairie Chickens. A very successful population that has genetic variation is the population of different finches on the Galapagos Islands that Darwin studied. The different birds adapted by growing different beaks for the different food sources so they wouldn’t compete for food. Also in some disease outbreaks within a population, individuals are affected differently because of their different genetic makeup. Some have major symptoms or even death, while others in the same population may experience nothing. All variations within a population can be modeled by the Hardy-Weinberg equation: p^2+2pq+q^2=1. The P^2 represents all the individuals within a population that have homozygous dominant alleles for the trait. The 2pq represents all the individuals within a population that have heterozygous alleles for the trait. The Q^2 represents all the individuals within a population that have the homozygous recessive alleles for the trait. This equation can also be represented in a punnett square which looks like the figure

to the right. The hereditary tree to the right shows how traits are passed down from generation.Multiple Choice Question: Which of the following are results of geneticvariation based on different environmental factors?I. Natural Selection II. The creation of antibiotic resistant pathogensIII. Two offspring of homozygous dominant parents for all alleles matingto make their own babiesIV. An Alaskan fox being able to change its coat color during seasonal changeA) I & 2 onlyB) III onlyC) All of the aboveD) I, II, IVFree Response Question: The Nazca Booby is a type of bird that is indigenous to the western parts of South America but more recently they have started to inhabit one of the islands of the Galapagos. Because the island the Boobies inhabit have no natural predators they have become more accustomed to people being around them. Name a few different phenotypic responses that the Booby could display while it is laying on its eggs and a human is approaching them for a stress test. Justify your answer with evidence from other animal species.

Page 16: 7 th /8 th  Review Project Collection

Answers

Multiple Choice:The answer to the multiple choice question is D). This is

because natural selection, the creation of antibiotic resistant pathogens and an Arctic Fox being able to change its coat color during the different seasons are all examples of genetic variation in response to different environmental factor. Natural selection will favor the animal that has the best genetics for its environment, like finches in the Galapagos changing their beak style so they wont compete for food. With antibiotic resistant pathogens the pathogen that isn’t effected by the drug will survive and reproduce to live on. Lastly the Arctic Fox is able to change its coat color because during the winter its environment is snowy and white so it needs a white coat for camouflage to hunt and in the warmer months its environment is brown so it changes its coat color to brown to camouflage itself during warmer months.

Free Response:Since the Booby has been isolated for so long from natural

predators the humans could have multiple effects on them. I believe the most common response to someone stressing out the bird on their nesting site would be to become territorial and try to attack the person. This is displayed in multiple animal like the lioness for instance when she is protecting her territory. The bird may also have no idea what to do and just sit there because it has never encountered something like this before. You can see this when dear stop in the middle of the road in oncoming traffic because they are just so frightened they have no idea what to do. A third response could be that the Booby would just turn around and pretend that the person isn’t there. A perfect quote can display this, “out of sight, out of mind.” These are three different phenotypic responses that a population of Booby could have to the same environmental factor.

Page 17: 7 th /8 th  Review Project Collection

Citations• http://www.google.com/imgres?imgurl=&imgrefurl=http%3A%2F%2Fbiology.westfield.ma.edu%2FBiol203%2Fpopulation-

genetics-outline&h=0&w=0&tbnid=m8pIe-HRxZf29M&zoom=1&tbnh=109&tbnw=143&docid=BqfPGbH2OZZA3M&tbm=isch&ei=dq1hU8DFFLSksQS22IHgAg&ved=0CAgQsCUoAg

• https://www.google.com/search?q=hardy+weinberg+equation&safe=off&tbm=isch&tbo=u&source=univ&sa=X&ei=RKxhU7e7JfTQsQT57YCgDg&ved=0CEoQsAQ&biw=1366&bih=685#q=genetic+variation+within+a+population&safe=off&tbm=isch&facrc=_&imgdii=_&imgrc=6_gppozc91-43M%253A%3Bkhqikx1KRVBpkM%3Bhttp%253A%252F%252Fevolution.berkeley.edu%252Fevolibrary%252Fimages%252Fnews%252Fpantheralleles.gif%3Bhttp%253A%252F%252Fevolution.berkeley.edu%252Fevolibrary%252Fnews%252F101201_panthers%3B487%3B274

Page 18: 7 th /8 th  Review Project Collection

LO 2.5: The student is able to construct explanations of the mechanisms and structural features of cells that allow organisms to capture, store or use free energy.SP 6.2: The student can construct explanations of phenomena based on evidence producedthrough scientific practices.Explanation: Free energy measures the portion of a system’s energy that can perform work when temperature and pressures are uniform throughout the system. Autotrophs capture free energy from the environment. Photosynthetic organisms capture free energy from sunlight. During photosynthesis chlorophylls absorb free energy from light which boast electrons to a higher energy level in photosystems I and II. Light energy is harnessed in photosystem here photons of light energy strike a chlorophyll molecule and excite electrons to a higher energy level. The thylakoid membrane is populated by photosystem I and photosystem II. When electrons are transferred between molecules in a sequence of reaction as they pass through the ETC a gradient of hydrogen protons across the thylakoid membrane is established. The ETC uses chemiosmosis to power ATP synthesize to make ATP; the special chlorophyll hits it again sending an electron to a higher energy level where it is captured by a different primary electron receptor. The electrons are passed along a shorted ETC and are transferred to NADP+ to create NADPH which is used in the Calvin Cycle. The Calvin Cycle occurs in the stroma of the chloroplast and releases NADP+ and ADP which is used in light reactions. Heterotrophs capture free energy present in carbon compounds produced by other organisms. Cellular Respiration is a set of process that makes energy for cells. The first step is glycolysis which means the splitting of sugar and it rearranges the bonds in glucose molecules which takes ADP and makes ATP and then the end product is pyruvate. Pyruvate is then transported across the mitochondria membrane by active transport for the Krebs cycle. In the Krebs cycle carbon dioxide is released from organic intermediates and ATP is synthesized from ADP and phosphate thorough substrate phosphorylation and electrons are captured by coenzymes. In the cycle it spins twice for each glucose molecule and one for each pyruvate molecule. NADH and FADH2 carry electrons to the electron transport chain. The ECT is in the mitochondria during cellular respiration and during photosynthesis it occurs in the chloroplasts. In cellular respiration electrons are passed to electrons acceptors as they move down the chain the final electrons acceptor is oxygen and in photosynthesis the final acceptor is NADP+.As the ETC accepts and then donate electrons they pump hydrogen ions form the mitochondrial matrix into the intermembrane space or in plants into the thylakoid membrane . Chemical energy is transformed into a proton motive force a gradient of H+ across the membrane. The hydrogen ions flow back down their gradient through a channel in an ATP synthase. The ATP synthase harnesses the proton motive force to phosphorylate ADP forming ATP. The use of a H+ gradient to transfer energy from redox reactions to cellular chemiosmosis. Together electron transport and chemiosmosis compose oxidative phosphorylation. All of these processes make free energy for organisms to use and expend in the environment.

18

Page 19: 7 th /8 th  Review Project Collection

19

MC Question: Tom has never had any energy to do the things he needs to do throughout the day. He is taken to the doctor for some test and they discover that his mitochondria produces more lactate than the average person. What is the best explanation for his problem?A.He does not have the transport protein that transports pyruvate across his mitochondriaB.His mitochondria is unable to perform cellular respiration C.His body can not make pyruvate in glycolysisD.He can not make ATP in his mitochondria

FRQ: A same plant species was observed in tow different lakes. The amount of oxygen dissolved was recorded through out the day. Using the graph and the data chart below and on the next slide compare the oxygen consumed in each lake and predict why there was a difference between the plants in each lake.

Page 20: 7 th /8 th  Review Project Collection

20

Data

Time Lake 1 O2 consumption Lake 2 O2 consumption6AM (0) .5 mg .9 mg8AM (1) 3.0 mg 2.2 mg10PM (2) 4.6 mg 3.4 mg12PM (3) 5.7 mg 4.8 mg2PM (4) 7.2 mg 6.4 mg4PM (5) 8.0 mg 7.0 mg6PM (6) 7.1 mg 6.2 mg8PM (7) 6.2 mg 5.3 mg10PM (8) 4.5 mg 3.1 mg12AM (9) 2.5 mg 2 mg

Page 21: 7 th /8 th  Review Project Collection

21

AnswersM.C : A. He does not have the transport protein that transports pyruvate across his mitochondria

A is the correct answer because since he produces so much lactate it is converted back to pyruvate but he does not have the transport protein that moves the pyruvate into the mitochondria to continue through cellular respiration in the citric acid cycle. Therefore his body does not produce ATP that gives him energy which explains why he is tired.

Free Response: Overall lake number 1 consumed more oxygen that lake 2. As the time went on in the day the oxygen consumption increased then reached its highest point in the afternoon. Then as the time passed the oxygen consumption decreased. The reason why both increased was because the sun was rising and it was getting hotter in the day as the sun set the oxygen consumption decreased. A reason why the plants in lake 1 has a higher oxygen consumption could be because it is exposed to more sunlight than lake 2. Which allows the plants in lake 1 to go thorough more photosynthesis which would explain why it has a higher oxygen consumption.

http://legacy.owensboro.kctcs.edu/gcaplan/bio/notes/BIO%20Notes%20E%20Photosynthesis.htm

Page 22: 7 th /8 th  Review Project Collection

LO 3.37 The student is able to justify claims based on scientific evidence that changes in signal transduction pathways can alter cellular response.SP 6.1 The student can justify claims with evidence.Explanation: A signal transduction pathway takes place during cell communication. Cells communicate by using neurotransmitters for short distances and hormones for long distances. The three stages of cell signaling include reception, transduction, and response. Reception is when a signal molecule (ligand) binds to the receptor protein on the outside of a target cell. Next, transduction (a series of events involving secondary messengers) occurs as the binding of the ligand causes a change in the receptor protein, which then brings about a specific cellular response. If there is a change in the signal transduction pathway, this can ultimately alter the cellular response, as the pathway in which the message is being carried along and amplified, controls whether or not the response occurs. Transduction occurs as a series of events, but it can also occur as a series of unfortunate events. Examples based on scientific evidence where pathways may be interrupted or altered include drugs, poisons, or diseases in the body. Poisons, such as those found in anthrax can cause a disruption in signal transduction pathways. Normally, a ligand would bind to a G-protein which then releases an alpha subunit that creates cAMP, which is a secondary messenger that amplifies the message and targets different parts within the cell. But when the poison that is found in anthrax is present, it targets the enzyme adenylate cyclase thus changing it’s shape, making it not able to convert ATP to cAMP. As a result, the signal transduction pathway is disrupted and the cellular response is altered. Another example is diabetes. Diabetes is a disease in which the body has shortage of insulin, a decreased ability to use insulin, or both. Insulin is a hormone that allows glucose to enter cells and be converted to energy. In each of our bodies there is an insulin receptor that sits on the surface of specific target cells. Once insulin binds to that receptor, a series of events occurs within the cell including an effect on the glucose transporter (GLUT) which allows the cell to take in glucose to make use of it by storing it as glycogen. With Type 1 Diabetes, a person is not producing insulin, therefore it doesn’t have the capabilities of binding with the receptor. With Type 2 Diabetes, the insulin receptor is ignoring the message that is trying to come through ultimately disrupting the signal transduction pathway by inactivating GLUT and as a result the person doesn’t take in glucose.

MC Question:If the insulin receptor located on a target cell was to become inhibited, what would most likely be the result of this disruption?(Look at diagram to the right)a)Insulin would still bind to the receptor protein but no message would be relayed.b)Insulin would diffuse through the cell membrane and target an intracellular receptor that was active.c)The amount of glucose would increase.d)The cells would eventually die due to the fact that there was no cellular activity (cell communication) taking place because the receptors were inhibited.

Free response: A group of scientists are designing an experiment in which they are trying to figure out how to inhibit a disruption in the signal transduction pathway via cell communication.a)Describe the process that occurs during normal cell communicationb)Discuss one thing that could help the inhibition of these pathwaysc)Provide evidence of a real life disease that results in a disruption to these pathways

Page 23: 7 th /8 th  Review Project Collection

MC Question:If the insulin receptor located on a target cell was to become inhibited, what would most likely be the result of this disruption?a)Insulin would still bind to the receptor protein and a message would be relayed.b)Insulin would diffuse through the cell membrane and target an intracellular receptor that was active.c)The amount of glucose would increase.d)The cells would eventually die due to the fact that there was no cellular activity (cell communication) taking place because the receptors were inhibited.

Free response: A group of scientists are designing an experiment in which they are trying to figure out how to inhibit a disruption in the signal transduction pathway via cell communication.a)Describe the process that occurs during normal cell communicationb)Discuss one thing that could help the inhibition of these pathwaysc)Provide evidence of a real life disease that results in disruption to these pathways

A is not correct because if insulin were to bind to a receptor that is inhibited then the message would not be relayed. B is not correct because insulin can’t diffuse through a cell because it is a protein (hormone) and it would be too big to pass through. D is not correct because cells don’t die because there isn’t any cellular activity, they die from programmed cell death or exposure to harmful environments. C is the answer because since the receptors are inhibited, insulin can’t bind which then can’t open the glucose transporter (GLUT) so the amount of glucose levels would increase since glucose can’t be taken into the cell for storage as glycogen.

Cell communication can occur through short or long distances. Cell communication starts out with reception when a ligand binds to a receptor, then transduction when the message is being amplified through a series of steps with the use of secondary messengers, and it finally results in a cellular response. One way that the scientists could inhibit this disruption is to inhibit the receptor in which the ligand binds to. If that is inhibited then no message can be relayed causing no disruption in the signal transduction pathway, also creating no cellular response. A real life disease in which this occurs is diabetes. With Type 1 Diabetes, a person is not producing insulin, therefore it doesn’t have the capabilities of binding with the receptor. With Type 2 Diabetes, the insulin receptor is ignoring the message that is trying to come through ultimately disrupting the signal transduction pathway by inactivating GLUT and as a result the person doesn’t take in glucose.

Page 24: 7 th /8 th  Review Project Collection

• LO 1.11: The student is able to design a plan to answer scientific questions regarding how organisms have changed over time using information from morphology, biochemistry, and geology. (See SP 4.2)

• SP 4.2: The student can design a plan for collecting data to answer a particular scientific question.

• Explanation: All organisms can be linked to other organisms in some way by common ancestry , and over a long period of time have branched off into their own species. Determining how organisms have evolved can be done by looking at physical features of the current organism and comparing them to those in fossils of the ancestor or to other organisms with similar structures. Depending on which rock layer a fossil is found in gives a rough estimate of when that organism lived, which helps scientists more easily place that species on a phylogenetic tree for comparisons and sequencing of ancestry. Genetic analysis shows similarities between organisms in terms of how many base pairs or amino acids are different. As time goes on, DNA/RNA sequences will change, leading to evolution of a species or the creation of a whole new species. To determine how organisms have changed over time, compare fossils or genetic sequences to see changes in physical structures or compare DNA sequences to see how mutations have led to evolution.

Page 25: 7 th /8 th  Review Project Collection

ABCD

• MC Question: Three fossils of now-extinct organisms are found: one in rock layer B, one in C, and one in D. Which one will have physical structures that most resemble those of an organism that thrives today? A) The one in rock layer BB) The one in rock layer CC)The one in rock layer DD) Cannot be determined

LL/FRQ Style Question: Assume that a particular species of frog and a particular species of lizard have a common ancestor, which is now extinct. Where the frogs and lizards live now was covered by water many hundreds of thousands of years ago, but now is mostly dried up.

A) Describe the best way to learn the physical structures of the common ancestor. B) Assume the common ancestor is a fish with two pectoral and two abdominal fins. When

the water dried up, what structural changes would you expect to see happening over a gradual period of time?Biochemistry involves the study of DNA sequences and genes. Morphology involves the study of the form and structure of organisms and their specific structural features.

C) Using evidence from either biochemistry or morphology, describe one way that you would find evidence to show how species change over time and become different than their ancestor. Describe how that evidence proves that a species has changed from its common ancestor.

Page 26: 7 th /8 th  Review Project Collection

• MC Question: Three fossils of now-extinct organisms are found: one in rock layer B, one in C, and one in D. Which one will have physical structures that most resemble those of an organism that thrives today? A) The one in rock layer BB) The one in rock layer CC)The one in rock layer DD) Cannot be determined

• A is correct because the fossil in layer B will be the youngest of the three, because the further down in the ground a fossil is, the older it is. Because the fossil in layer B is the youngest, it will be most closely related to an animal that lives today, meaning that their structures will be the most similar.

LL/FRQ Style Question: • A) The best way to learn the physical structures of the common ancestor are to look at fossils of it.

Fossils provide a depiction of what exactly the organism looked like, and therefore, we will learn the structures the organism had based on what structures we see in the fossil.

• B) We would expect to see the fins gradually take the shape of legs. We know that lizards and frogs have legs, and they need them to travel on land. As the water dried up, there was no need to have fins anymore, and organisms with structures allowing them to be more mobile on land would be favored. Also, the four fins are already in an ideal place to become legs.

Page 27: 7 th /8 th  Review Project Collection

• C) Biochemistry: To find evidence of how the species has changed over time, sequence the DNA of the ancestor and the present day species that is a descendant of the ancestor. By finding differences in the sequences of their DNA, it can be determined what genes are different than the ancestor. By having differences in the sequencing of DNA, we can see how exactly on a molecular level the species has changed over time.ORMorphology:To find evidence of how the species has changed over time, examine the structures of the current day animal and the ancestor by using fossils, and make observations of the similarities and differences. By finding different physical features, we know that the species has changed over time from its ancestor. Similarities in form will show ancestry, but there still should be some differences in structure as some structures are favored over others as conditions change.

This picture shows similar structure in four different organisms. We know by the similar structure that there is common ancestry involved, however, the structures are slightly different. The common ancestor must have encountered some environmental pressure that made it need different functions for its limbs, leading to a physical change in structure.

Page 28: 7 th /8 th  Review Project Collection

LO 4.9 The student is able to predict the effects of a change in a component(s) of a biological system on the functionality of an organism(s).

SP 6.4 The student can make claims and predictions about natural phenomena based on scientific theories and models.

Explanation: A Biological system is one uses free energy to maintain equilibrium, reproduce, and grow. They further use different means to obtain and utilize free energy and other resources. The goal outlined by Learning Objective 4.9 is to allow for the student to better understand the effects that are associated with changes to components of living systems, and how any changes would effect the ways in which the organism functions and adapts accordingly. Using SP 6.4, a student should be able to draw parallels between different scientific evidence to assess potential results that would occur with these changes.

MC Question:What effects upon an organism would occur if it has contracted diabetes and the pancreas is no longer

able to produce any or sufficient insulin?a) The blood sugar levels of the organism would decrease.b) The blood cells would become turgid due to the lack of homeostasis.c) The blood sugar levels of the organism would increase.d) There would be increased blockage in the organism’s arteries. Learning Log/FRQ:Describe how the Human Immunodeficiency Virus attacks the components of an organism and affects the

functionality of the immune system? Identify what type of virus the HIV virus is and describe how it reproduces itself in the host biological system. Assess why a multitude of AIDS patients ultimately die of common illnesses.

Page 29: 7 th /8 th  Review Project Collection

LO 4.9/ SP 6.4 (Answers)

• Multiple Choice Answer:– (C) is the correct answer because the lack of insulin would make it hard for the body to

break down sugars, leaving higher glucose concentration in the blood, which would make (A) incorrect. (B) is incorrect because the insulin or lack of insulin has no effect on the concentration of the blood cells, which affect turgidity, and (D) is incorrect because the diabetes has no correlation to increase of plaque in arteries

• Learning Log/FRQ AnswerThe HIV virus attacks important immune system cells known as helper-T cells that lyses

infected cells, so without these helper-T cells, the immune system would not work properly and leave the individual to be vulnerable to diseases. . The HIV Virus is a type of retrovirus that reproduces by binding to a blood cell and injecting the nucleic acid and reverse transcriptase into the cell that will translate the RNA into DNA and transport it to the nucleus of the cell where it will be integrated with the DNA of the host cell. Thus the cell creates more HIV cells and once it reaches full capacity the blood cell lyses and releases the reproduced HIV cells that can repeat the process on other blood cells. Once this stage has occurred, the organism has contracted AIDS, and their immune system is heavily compromised. Thus, they are left vulnerable to diseases, and could potentially die of common illnesses due to the helper T-cells being unable to properly carry out their jobs.

Page 30: 7 th /8 th  Review Project Collection

LO 2.24: The student is able to analyze data to identify possible patterns and relationships between a biotic or abiotic factor and a biological system (cells, organisms, populations, communities or ecosystems).

SP 5.1: The student can analyze data to identify patterns or relationships.Explanation: In regards to cell activity, for example, a plant would lose some of its water through evapotranspiration if it’s exposed to too much sunlight. (If the data were

recorded it would show how as light exposure increased so would the rate of evapotranspiration.) In contrast, if a short night plant isn’t exposed to sufficient sunlight it won’t flower since the night wasn’t shorter than a critical duration.(Data would show low flowering rates in relation to low amounts of light exposure.) Also, a plant cell will produce abscisic acid when it’s cold (low temperatures) which will inhibit the growth of this organism. (Low temperatures would then correlate with decreased growth of individuals.) An organism’s activities are also affected by its interactions with biotic and abiotic factors. Such an example would be that when temperatures get severely low (long, cold winters) some animals hibernate; woodchucks’ heartbeats go from 80 to 4 or 5 beats per minute, also dropping body temperatures 60 degrees (oF) below normal. (If data were to be recorded it would show the relationship between cold weather and the hibernation patterns of animals.) The stability of populations, communities, and ecosystems is also affected by interactions with factors such as water availability, food chains, and population density. Such as how most plant populations won't thrive during droughts (Decreased water becoming correlated with the decrease of those plant populations.), or how more competition may lower the survival rate of a certain species (Increased population of one species decreasing that of another.).

M.C. Question: Based on the data provided which can be inferred?

I. The abiotic factor has no association/correlation with the size of the population.II. Crop production increases as annual precipitation increases.III. The biological system presented in this question affects the abiotic factor shown.

A) Only IIIB) II and IIIC) I, II, and IIID) Only IIE) I and II

Slide 1 of 4

Page 31: 7 th /8 th  Review Project Collection

Learning Log/FRQ-style Question: Biological systems such as populations are affected by both biotic and abiotic factors.

a. Identify two abiotic factors that could cause the fluctuations seen in the graph provided.

i. Elaborate on one of the abiotic factors you identified.

b. Identify two biotic factors that could cause the fluctuations seen in the graph provided.

i. Elaborate on one of the biotic factors you identified.

Slide 2 of 4

Page 32: 7 th /8 th  Review Project Collection

ANSWER KEY-LO 2.24

Based on the data provided which can be inferred?

I. The abiotic factor has no association/correlation with the size of the population.II. Crop production increases as annual precipitation increases.

III. The biological system presented in this question affects the abiotic factor shown.

A) Only IIIB) II and IIIC) I, II, and IIID) Only IIE) I and II

-------------------------------------------------------------------------------------------------------------------------------------------------------------------- A) III is wrong since the biological system (population-size) is the response variable not the explanatory one.

B) Can’t be right since it includes III (see A).C)Can’t be right since it includes III and I (see A for III, and E for I).D) II is right since as X (annual precipitation) increases then Y (crop production) increases.E) Can’t be right since it includes I; I is wrong since there is an association evident between the explanatory variable (annual precipitation-abiotic factor) and the response variable (population size) (positive slope seen).

Slide 3 of 4

Page 33: 7 th /8 th  Review Project Collection

Biological systems such as populations are affected by both biotic and abiotic factors.a. Identify two abiotic factors that could cause the fluctuations seen in the graph provided.

i. Elaborate on one of the abiotic factors you identified.b. Identify two biotic factors that could cause the fluctuations seen in the graph provided.

i. Elaborate on one of the biotic factors you identified.

a. One abiotic factor that could cause these fluctuations is rain, which affects water availability. Such as how some years there are droughts and during others there’s plenty of water available and isn’t a concern to the populations found within a given area. Another abiotic factor could be wind. Such as how wind helps/aids in the dispersion of dandelion seeds. If winds aren’t strong enough during a pollination season it would decrease the population of the next generation.i. During a period of drought the population size of a grass would decrease due to lack of water. After the period of drought the population would once again flourish and be abundant in its location of growth. Due to all the land that was available too much grass flourished depleting the amount of nutrients in the soil causing the population to once again decrease. Then once there’s a good amount of rain in a certain period that would help restore some of the nutrients in the land. Changes would continue to happen in this manner going from plenty of individuals to the population number dropping again.

b. One biotic factor that could cause the fluctuations seen would be the amount of plants available. Such as not enough grass being able to maintain the current population of rabbits. Another biotic factor would be an animal. Such as there being more of a predatory population than the prey population, affecting one another. i. During a year when the population of rabbits(the prey) increases the population of wolves would then increase as well (the predator). The wolf population would then exceed its carrying capacity by overhunting the rabbit population of rabbits, decreasing one of their resources. Then the wolf population would decrease due to starvation/competition for resources, allowing for the population of rabbits to once again be restored. This increase and decrease (the fluctuations observed on the graph) in wolf population would continue in this manner.

Slide 4 of 4

Page 34: 7 th /8 th  Review Project Collection

Learning Object 2.36: The student is able to justify scientific claims with evidence to show how timing and coordination of physiological events involve regulation. Science Practice 6.1: The student can justify claims with evidence. Explanation: The timing and coordination of physiological events like hibernation, estivation, circadian rhythms, and such are in response to internal and external cues both environmental and systemic. Physiological events occur in tandem with environmental changes, like hibernation during the winter or a nocturnal species sleeping during the daylight. In hibernation, species go into torpor, or a low activity state with decreased metabolism, which allows them to conserve energy in times where temperatures are extreme or food is not easily available. Squirrels, for example, provide evidence of this situation: in the winter the Belding’s ground squirrel drops body temperature from 37 C to close to freezing and regulates its metabolism down to about 5-8 kcal per day from around 85 kcal per day. Plants respond mainly to the stimuli of phototropism and photoperiodism or the response to light or length of light, which generally corresponds to certain times during each day. Multiple Choice: Which of the following scenarios is not an example of regulation of physiological events by timing or coordination?A.Plants like clover and carnations are long-day plants while hemp and cotton are short-day plants based on the length of the nightB.Bees dance in specific patterns to alert fellow bees where food can be foundC.A biological clock triggers stomata to open and close in an approximately 24 hour cycleD.Bacteria use quorum sensing based on density to organize antibiotic resistance, virulence, and biofilm formation

Free Response Question: A study was done on a representative group of ground squirrels whose body temperature and metabolic rates were measured for one month. The results are summarized in the graph to the right. The dashed line represents the mean metabolic rate for the ground squirrels during the summer time when they do not hibernate. Considering the diagram to the right, answer the following questions:a.Describe the graph and explain what the changes in body temperature and metabolic rate indicate in terms of hibernation.b.Explain the evolutionary advantage of hibernation for ground squirrels. Justify your conclusions using evidence from the graph.c.Identify and describe the method of regulation of metabolic rate during hibernation and what triggers it.

http://ajpregu.physiology.org/content/ajpregu/302/1/R15/F1.large.jpg?width=800&height=600

Page 35: 7 th /8 th  Review Project Collection

Multiple Choice: Which of the following scenarios is not an example of regulation of physiological events by timing or coordination?A.Plants like clover and carnations are long-day plants while hemp and cotton are short-day plants based on the length of the nightB.Bees dance in specific patterns to alert fellow bees where food can be foundC.A biological clock triggers stomata to open and close in an approximately 24 hour cycleD.Bacteria use quorum sensing based on density to organize antibiotic resistance, virulence, and biofilm formation

B is the correct answer. Choice A involves the regulation of plant flowering based on the critical darkness periods that the plant receives. Choice C is an example of a circadian rhythm triggering stomata to open and close based on the daytime in response to transpiration. Choice D is bacteria responding to density based cues to coordinate a response to various stimuli. The figure below demonstrates this effect. The triangles represent quorum sensing molecules that are a form of communication between bacteria. Choice B, however, is a reference to the studies of Karl von Frisch who observed the manners in which honey bees communicate to other bees.

Free Response Question: A study was done on a representative group of ground squirrels whose body temperature and metabolic rates were measured for one month. The results are summarized in the graph to the right. The dashed line represents the mean metabolic rate for the ground squirrels during the summer time when they do not hibernate. Considering the diagram to the right, answer the following questions:a.Describe the graph and explain what the changes in body temperature and metabolic rate indicate in terms of hibernation.b.Explain the evolutionary advantage of hibernation for ground squirrels. Justify your conclusions using evidence from the graph.c.Identify and describe a possible method of regulation of metabolic rate during hibernation and what triggers it.

Sample Answer: a. The graph has three peaks that reach about 40 degrees C and a metabolic rate of 5 mL O2 h-1 g-1 that last for very short periods of time around a few hours. Between these spikes are much lower fluctuating periods with a small variability. These areas stay rather steady at 4 degrees C and oscillate in metabolic rate between above 0 and 1 mL O2 h-1 g-1 . The lower extended periods last about ten minutes and represent the torpor stage of the hibernation. In this period, the core body temperature lowers such that all protein activity slows and the overall metabolic rate slows. The spikes indicate the animal waking up to complete maintenance type functions that require a higher metabolic rate before returning to the torpor.

http://1.bp.blogspot.com/-9AB7B7u_JqM/TdMrztsRrCI/AAAAAAAAJMw/k0wKULTSMKg/s1600/bob1.jpg

b. Hibernation is an adaptive evolutionary trait that allows ground squirrels a better chance to survive and pass on the trait. During hibernation, the squirrels can conserve a large amount of energy by existing in a low metabolic rate state. The metabolic rate is the sum of all chemical reactions occurring within the organism, many of which require ATP. In the winter there food is less available due to the cold and therefore minimized opportunities to obtain glucose to power cellular respiration to produce ATP. Further, more energy would be required for squirrels to maintain their usual high core temperature during the cold because of their endothermic nature. The total area under the graph of the metabolic rate for the hibernation is much less than the area underneath the mean summer metabolic rate, which lies at about 1.3 O2 h-1 g-1, much higher than the around 0.2 O2 h-1 g-

1 norm for hibernation. For the aforementioned reason of cold temperature requiring more energy to maintain body heat, the mean metabolic rate for winter without hibernation may even be higher than that for summer when there is already heat. This overall conservation of energy allows the ground squirrel a much better chance of surviving the winter and reproducing. c. Hibernation is triggered by a prolonged change in temperature. Periods of cold weather slow down the body’s metabolic processes by lowering kinetic energy and thus overall energy and reactions. Thus, in hibernation, animals like ground squirrels allow their body temperatures to drop very low such that the metabolic rate is greatly slowed as kinetic energy takes a sharp decline. As seen in the graph, temperature and metabolic rate are closely linked such that this explanation can be backed that the lowering of temperature leads to the slowing of the metabolic rate.

Page 36: 7 th /8 th  Review Project Collection
Page 37: 7 th /8 th  Review Project Collection

MC: Which of these conditions indicate positive feedback has occurred?A) Vigorous exercise raises body temperatures above normal, and sweat is produced.B) Once a vessel is damaged, platelets start to cling to the injured site and release chemicals that attract more platelets.C) When the body temperature drops, blood vessels constrict, and shivering begins.D) A decrease in oxygen causes the kidneys to secrete erythropoietin, a hormone that stimulates the production of red blood cells.

B is the correct answer because it is the only answer choice that demonstrates positive feedback. With blood clotting, the platelets continue to pile up and release chemicals until a clot is formed, thus initiating an amplified response. Answer choices A, C, and D are all examples of negative feedback. We know this because in each case, a change in the variable being monitored triggers the control mechanism to counteract further change in the same direction. When the body temperatures are above normal or below normal, sweat or shivering is the response that is used to reverse the output. Same thing with D, if there is a lack of oxygen, a hormone that stimulates the production of red blood cells is secreted.

FR: Distinguish the difference between positive feedback and negative feedback, and provide examples for each. Why are feedback loops important?With negative feedback, the output reduces the original effect of the stimulus. In a positive feedback system, the output enhances the original stimulus. An example of negative feedback is the control of blood sugar with insulin. When blood sugar rises, receptors in the body sense a change . In turn, the pancreas secretes insulin into the blood effectively lowering blood sugar levels. Once blood sugar levels reach homeostasis, the pancreas stops releasing insulin. An example of positive feedback is lactation. As the baby suckles on the nipple there is a nerve response into the spinal cord and up into the hypothalamus of the brain, which then stimulates the pituitary gland to produce more prolactin to produce more milk. Feedback loops are important in ensuring the receptor will not continue to stimulate the effector, leading to an overcorrection and causing a deviation in the opposite direction.

Image courtesy of http://biology-forums.com/index.php?action=gallery;sa=view;id=1077

Page 38: 7 th /8 th  Review Project Collection

LO 1.10 The student is able to refine evidence based on data from many scientific disciplines that support biological evolution. SP 5.2: The student can refine observations and measurements based on data analysis.

Explanation: Natural selection is a major component in supporting biological evolution. Natural selection was exposed broadly by Charles Darwin. Natural selection is the theory that states the more favorable a phenotype is more likely it is to survive and pass the gene through offspring. Also another piece of evidence would be homologous structures. Homologous structures are structures that have various development and structures with different functions. Analogous structures are the complete opposite these organisms have the same function with no trace of a common ancestor. These biological theories can be measured and refined through a series of process such as analysis of cladograms, analysis of phylogenetic trees and measurements of fitness(reproductive success).

M.C. Based on the cladogram provided which specie has the most recent common ancestor to the Rodents and Rabbits ? ( next page)A. BirdsB. Amniotic EggC. PrimatesD. SharksF.R. / L.L: Based on your knowledge of biological evolution explain the following:A.) Compare & contrast divergent and convergent evolution and its significance toBiological evolution.

Page 39: 7 th /8 th  Review Project Collection
Page 40: 7 th /8 th  Review Project Collection

Answer KeysM.C. Based on the cladogram provided which specie has the most recent common ancestor to the Rodents and Rabbits ?A. BirdsB. Amniotic EggC. PrimatesD. SharksC: Is the correct answer because the Primates and Rabbits are branched off from each

other. This branch is a symbolizing the common ancestor.

Compare & Contrast divergent and convergent evolution and its significance to biological evolution:Divergent evolution is one specie that develops into two species over time. For example the Wooly Mammoth(extinct) that developed into the common Elephant today. The newly created specie live in different ways from the common ancestor. They are created through migration or near extinction. Convergent evolution is when two different unrelated organisms develop common features because, of adapting in similar environments. This is is significant in biological evolution because it creates biological variation and ensures that each species plays out its niche.

Answer Keys

Page 41: 7 th /8 th  Review Project Collection

LO 3.34: The student is able to construct explanations of cell communication through cell-to-cell direct contact or through chemical signaling.SP 6.2: The student can construct explanations of phenomena based on evidence produced through scientific practices.Explanation: The process of cell communication is primarily executed via direct contact between two cells, mainly through means of surface receptors located on the outer membranes of cells, which, upon activation by a stimulus, subsequently trigger the signal transduction pathway within the recipient cell to trigger a response. Direct communication may also transgress via gap junctions or plasmodesmata that allow materials to be transported between animal and plant cells, respectively. Another form of cellular communication involves that of chemical signaling, which functions similarly to direct contact, save for the use of secreted chemical signals rather than membrane-integrated signal molecules to perform communicative tasks, such as with the intracellular activation of responses from chemical signals, localized synaptic signaling between neurons, and more long-distance hormonal signaling between an endocrine cell and its respective target cell.

MC Question: Cell surface receptors consist of each of the following, excluding:a) G protein linkedb) Enzymatic receptorsc) Single-pass transmembrane proteinsd) Chemically-gated ion channelsFRQ Question: The endocrine systemutilizes cell-to-cell communication to initiate responses upon any internal or external environmental change. Describe each of the three pathways that make up hormonal distribution in the body, taking note of specific chemical signals and receptors used in each to carry out cellular communication.

Page 42: 7 th /8 th  Review Project Collection

MC Question: Cell surface receptors consist of each of the following, excluding:a) G protein linkedb) Enzymatic receptorsc) Single-pass transmembrane proteinsd) Chemically-gated ion channelsFRQ Question: The endocrine system utilizes cell-to-cell communication to initiate responses upon any internal or external environmental change. Describe each of the three pathways that make up hormonal distribution in the body, taking note of specific chemical signals and receptors used in each to carry out cellular communication.

The simple endocrine pathway, in response a stimulus, triggers a receptor protein on an endocrine cell, which subsequently releases hormones into a blood vessel to the corresponding target cell, triggering the response. The simple neurohormone pathway, however, upon stimulation, triggers the activation of either the hypothalamus or posterior pituitary via sensory neurons, causing the release of neurohormones by a neurosecretory cell. The neurohormones then travel though a blood vessel to the target cells, triggering a response. Both simple endocrine pathways and simple neurohormone pathways appear present in the simple neuroendocrine pathway, which involves the stimulation or sensory neurons that activate receptors in the hypothalamus, signaling the release of neurohormones into a blood vessel, which then transport them to receptor sites on an endocrine cell. The cell subsequently releases hormones into another blood vessel that are then transported to the target cell, triggering the response.

Page 43: 7 th /8 th  Review Project Collection

LO 3.21 The student can use representations to describe how gene regulation influences cell products and function.SP 1.4 The student can use representations and models to analyze situations or solve problems qualitatively and quantitatively.

Explanation The products and function of genes are the results from gene expression. While some genes are continually expressed, the expression of most genes are regulated. Regulation allows more efficient energy utilization, allowing an increase in metabolic fitness. Regulatory genes are able to control gene expression through environmental signals and developmental cascades. One of the best studied regulatory systems is the inducible and repressible regulatory systems that use operons in bacteria. Operons are made up of a promoter, an operator, and the genes they control. The operator acts as an “on, off” switch, which is controlled by a protein repressor. The repressor protein, when active, binds to the operator and switches the operon off by blocking RNA polymerase from transcribing the DNA into RNA. A repressible operon is an operon that has the transcription on but can only be inhibited when a corepressor molecule allosterically binds to the repressor protein and makes it active. An inducible operon is an operon that has the transcription off but can only be stimulated when an inducer interacts with repressor protein and makes it inactive.

M.C. Question:The diagram above is one of an operon. Identify what type of operon it is AND what is needed in order for transcription to happen.A.An inducible operon which a corepressor molecule will make the repressor protein inactive and allow transcription to occur.B. An inducible operon which an inducer molecule will make the repressor protein inactive and allow transcription to occur.C. A repressible operon which a corepressor molecule will make the repressor protein active and allow transcription to occur.D.An inducible operon which a inducer molecule will make the repressor protein active and allow transcription to occur.

Learning Log/ FRQ-style Question: The diagram above is one of a trp operon that produces polypeptides that make up enzymes for tryptophan synthesis. The operon is inhibited when Trytophan binds to the repressor protein and the active repressor protein binds to the operator. An enzyme called Tryptophandestructase can also bind with tryptophan and denatures it. Suppose Tryptophandestructase was introduced, explain what will happen to the operon. Justify your answer.

Page 44: 7 th /8 th  Review Project Collection

LO 3.21 The student can use representations to describe how gene regulation influences cell products and function.SP 1.4 The student can use representations and models to analyze situations or solve problems qualitatively and quantitatively.

Sample Answer: Because Tryptophan makes the repressor protein active, tryptophan is a corepressor, this would make the trp operon a repressible operon. Because the operon is repressible, it will continue to function unless the corepressor, tryptophan, binds with the repressor and inhibits the operon. Since tryptophan is being denatured by Tryptophandestructase, the repressor protein will remain inactive, the trp operon will be uninhibited, and the operon will continue to produce polypeptides that make up enzymes for tryptophan synthesis.

M.C. Question:The diagram above is one of an operon. Identify what type of operon it is AND what is needed in order for transcription to happen.A.An inducible operon which a corepressor molecule will make the repressor protein inactive and allow transcription to occur.B. An inducible operon which an inducer molecule will make the repressor protein inactive and allow transcription to occur.C. A repressible operon which a corepressor molecule will make the repressor protein active and allow transcription to occur.D.An inducible operon which a inducer molecule will make the repressor protein active and allow transcription to occur.

Learning Log/ FRQ-style Question: The diagram above is one of a trp operon that produces polypeptides that make up enzymes for tryptophan synthesis. The operon is inhibited when Tryptophan binds to the repressor protein and the active repressor protein binds to the operator. An enzyme called Tryptophandestructase can also bind with tryptophan and denatures it. Suppose Tryptophandestructase was introduced, explain what will happen to the operon. Justify your answer.

Slide by Alex VarsikInducible operon diagram from http://www.biologyjunction.comTrp operon from Campbell Reece Ap Biology Textbook 7th Edtion.

Page 45: 7 th /8 th  Review Project Collection

LO 4.19:The student is able to use data analysis to refine observations and measurements regarding the effect of population interactions on patterns of species distribution and abundance.

SP 5.2: The student can refine observations and measurements based on data analysis.

• Explanation: Interactions between populations affect population distribution and abundance because of processes such as competition, predation, and mutualism. The population interactions can have positive effects, where the population change is beneficial, or negative effects, where the change is hurtful. Populations may move out of an area to find more beneficial conditions, which changes both the old and new ecosystems with the loss or gain of a population. This can mean that one ecosystem gains a new species, but loses another because of over predation. Feedback controls play a role in how population changes affect symbiotic relationships within an ecosystem because if one animal preys on another, the preys population size decreases, the predators will not have a large enough food source for their entire population, and their population size will decrease. This shows that there is a very delicate balance between populations. If one population in an area changed in density or dispersion, this would dramatically change many other populations in the area because populations depend on each other.

1/5

Page 46: 7 th /8 th  Review Project Collection

• M.C Question:The graph shown here depicts the numbers of moose and wolf populations in an area. Infer from the graph what caused the spike in moose population in the years leading up to 1995.

• a) The increase in number of wolves meant that they had to find other prey, allowing more moose to survive.

b) The wolf population was smaller than normal, causing there to be less predators for the moose and more could survive.c) There were extremely good environmental conditions during the moose mating season.d) The moose developed an adaptation that allowed them to escape the wolves.

2/5source: http://p4stsbio2010.blogspot.com/2010_09_01_archive.html

Page 47: 7 th /8 th  Review Project Collection

Source of graph: http://myweb.rollins.edu/jsiry/LeolopoldMountainMeans.html •

3/5

• Learning Log/FRQ-style Question: In the graph below, it shows the population changes of a hare and a lynx. Assuming that the lynx is a predator to the hare, answer the following questions about the populations.

a) Explain in detail the population changes and why there are peaks and dips in the population size.

b) Are the hare and lynx showing positive or negative feedback? How do you know?

c) Supposing that the hare is the main food source for the lynx, what would happen to each population if the hare developed an adaptation that protected it from the lynx. What effects would this have on the ecosystem?

Page 48: 7 th /8 th  Review Project Collection

Answers Key - LO 4.19• M.C Question:The graph shown here depicts the numbers of moose and wolf populations in an area. Infer from the graph

what caused the spike in moose population in the years leading up to 1995.

• a) The increase in number of wolves meant that they had to find other prey, allowing more moose to survive.

b) The wolf population was smaller than normal, causing there to be less predators for the moose and more could survive.c) There were extremely good environmental conditions during the moose mating season.d) The moose developed an adaptation that allowed them to escape the wolves.

We know the answer is B because in the graph, you can see there is a sharp increase in the moose population at the same time the wolf population is at its lowest points. This means that more moose can survive because there is less threat of predators.

4/5

Page 49: 7 th /8 th  Review Project Collection

Answers Key - LO 4.19

• In the graph below, it shows the population changes of a hare and a lynx. Assuming that the lynx is a predator to the hare, answer the following questions about the populations.

a) Explain in detail the population changes and why there are peaks and dips in the population size.b) Are the hare and lynx showing positive or negative feedback? How do you know?c) Supposing that the hare is the main food source for the lynx, what would happen to each population if the hare developed an

adaptation that protected it from the lynx. What effects would this have on the ecosystem?

a) The populations of the hare and the lynx are connected, and change together. When the hare’s population increases, so does the population in the lynx because they have more of their food source, causing a peak in the populations. The predation of the hare from the increased lynx population results in the decrease of the number of hare, also causing a dip in the lynx populations.

b) The hare and the lynx are showing positive feedback because they move in the same direction. When one population goes up, so does the other and vice-versa.

c) If the hare adapted something to protect it from the lynx, than the hare population would increase without predation while the lynx population decreased because of their loss of food supply. In the ecosystem, this could have disastrous effects because the hares would need more food to feed their growing population, and without a proper check on the hare population, their population would consume all the food in the area, disrupting the entire food chain.

5/5

Page 50: 7 th /8 th  Review Project Collection

Learning Objective 2.1LO 2.1: The student is able to explain how biological systems use free energy based

on empirical data that all organisms require constant energy input to maintain organization, to grow and to reproduce.

SP 6.2: The student can construct explanations of phenomena based on evidence produced through scientific practices.

We can tell organisms use energy and need energy for metabolic processes because of the exchange between the different trophic levels. The law of thermodynamics, which states that matter cannot be created or destroyed, helps explain how energy is used for organisms. Organisms obtain energy from other organisms and use that energy in energy coupling reactions. This means that there must be a constant energy input in order increase or decrease the entropy. There is a transfer of energy by photosynthesis or chemosynthesis from free energy reactions. There is evidence of this energy transfer in metabolic rates, physiological changes, variations in reproductive strategies, and effects of changes in free energy availability. If there is an offset of free energy availability for a clover population, a change in population size can be seen in that autotrophic population as well as a rabbit population (heterotrophic) that uses the clovers for energy.

M.C. Question: If someone standing still on the edge of a diving board was pushed off the board into a pool below the board, which of the following is true?

I. The diver will move toward a more stable state the closer to the water they getII. The free energy of the diver would increase the closer to the water they getIII. The movement of the diver from the board to the water is a spontaneous reaction that

releases free energya) I onlyb) II onlyc) II and III onlyd) I and III onlyFRQ: Using the figure, describe where each organism is obtaining energy, two chemical

processes responsible for the harnessing of energy, and what happens to the energy as it is transferred from each trophic level.

Page 51: 7 th /8 th  Review Project Collection

Answer KeyM.C. Question: If someone standing still on the edge of a diving board was pushed off the board into a pool below the board, which of the following is true?

The diver will move toward a more stable state the closer to the water they get

The free energy of the diver would increase the closer to the water they get

The movement of the diver from the board to the water is a spontaneous reaction that releases free energy

a) I only

b) II only

c) II and III only

d) I and III only

FRQ: Using the figure, describe where each organism is obtaining energy, two chemical processes responsible for the harnessing of energy, and what happens to the energy as it is transferred from each trophic level.

The primary producers, or the plants, are obtaining energy from the sun using photosynthesis. During photosynthesis, light energy is converted into chemical energy through a network of photosystems in the chloroplasts of leaves. Light strikes the thylakoids in the chloroplasts to excite electrons to a higher level. Once a certain level is met, the electrons initiate photosynthesis. The light reactions split water and oxygen is given off from the Calvin cycle. During the Calvin cycle, carbon fixation, reduction reactions, and the regeneration of RuBP occurs. The Calvin cycle uses enzymes such as RuBisCo to break down carbon molecules into two molecules of 3-phosphoglycerate(3PGA), then ATP is used to phosphorylate 3PGA to form ADP. The reduction reactions use the NADPH from the light reactions to form G3P and NADP+. The rabbit population obtains energy from the plants; the snake population from the rabbit population; and the eagle population from the snake population. As the energy is transferred up the trophic levels, the same amount of energy is given off as heat into the ecosystem.

Page 52: 7 th /8 th  Review Project Collection

LO 3.20: The student is able to explain how the regulation of gene expression is essential for the processes and structures that support efficient cell function.

SP 6.2: The student can construct explanations of phenomena based on evidence produced through scientific practices.

Explanation: Gene expression is regulated at the DNA, RNA, and protein levels. At the DNA level histone acetylation patterns and DNA methylation patterns control which genes can be transcribed. DNA with a methyl group (-CH3 ) or with unacetylated histone tails can not be transcribed. At the RNA level enhancers or silencers are transcription factors that can bind to the DNA near the promoter to activate or suppress specific genes. Transcription factors regulatory proteins that are required for transcription but silencers can also halt transcription. Alternative mRNA splicing can create different mRNA strands using regulatory proteins during RNA processing. MicroRNA (miRNA) is used to degrade mRNA. It binds to mRNA to make it double stranded. The double stranded mRNA can not be translated and it is selected to be degraded into nucleotides. At the protein level ubiquitin bunds to a protein that is no longer needed. The ubiquitin signals for the protein to be broken down in a proteasome into its amino acids. The regulation of gene expression causes many phenotypic differences seen in organisms with similar DNA sequences.

MC: Why are red blood cells and muscle cells different if they have the same DNA?A They have different genes even though they have the same DNAB They have different activators in their nucleus C Red blood cells are produced from bone marrow but muscle cells are notD The genes have different promoters in each cell.

Slide 1 of 3

http://bio1151.nicerweb.com/Locked/media/ch18/euk-regulation_cytoplasm.html

Page 53: 7 th /8 th  Review Project Collection

FRQ: Predict the result of each scenario and explain it in terms of gene regulationa. A eukaryotic cell is treated with a chemical that acts as an enhancer for a cell division promoting gene.b. A eukaryotic cell is treated with a chemical that removes the methyl group from DNA.c. A eukaryotic cell is treated with a chemical that binds to miRNA disabling it.

Slide 2 of 3

Page 54: 7 th /8 th  Review Project Collection

MC: Why are red blood cells and muscle cells different if they have the same DNA?A They have different genes even though they have the same DNA

-A is incorrect because the cells have the same DNA so they have the same genes.B They have different activators in their nucleus

-B is correct because the variation of activators means that different genes are transcribed in each cell.C Red blood cells are produced from bone marrow but muscle cells are not

-Though C is true it does not answer the question.D The genes have different promoters in each cell

-D is incorrect because the cells have the same DNA sequence so the same promoters for each gene are in the same spot in each cell

FRQ: Predict the result of each scenario and explain why in terms of gene regulation.a. A eukaryotic cell is treated with a chemical that acts as an enhancer for a cell division promoting gene.b. A eukaryotic cell is treated with a chemical that removes the methyl group from DNA.c. A eukaryotic cell is treated with a chemical that binds to miRNA disabling it.

A- The cell will continue to divide because an excess of the dividing protein is present. The chemical works as a transcription factor promoting the transcription of the gene instead of it being regulated. The mRNA encoding for the protein will be translated to create the protein to promote cell division. This could lead to a tumor. B- Most of if not all the DNA will now be able to be translated causing a build up of proteins that are not needed. Methylated DNA can not be translated because RNA Polymerase can not function if it is blocked by a methyl group, thus preventing unneeded proteins from being made. With the methyl groups removed, the DNA will be transcribed then the mRNA translated creating excess unnecessary proteins.

C- mRNA will not be degraded causing excess proteins the cell is currently making to collect. miRNA binds to the mRNA signaling for the degradation of it. If the miRNA are unable to bind to mRNA it will not be broken down therefore it can not be regulated, and it can only be translated creating more protein.

Slide 3 of 3

Page 55: 7 th /8 th  Review Project Collection

LO 3.47 The student is able to create a visual representation of complex nervous systems to describe/ explain how these system detect external and internal signals, transmit and integrate information, and produce responses.SP 1.1 The student can create representations and models of natural or man-made phenomena and systems in the domain.

ExplanationThe Peripheral Nervous System, which contains sensory and motor system that are responsible for the input and output by relaying information to the central nervous system. The neuron is the basic structure of the nervous system that reflects function. A typical neuron has a cell body, axon and dendrites. Many axons have a myelin sheath that acts as an electrical insulator. The structure of the neuron allows for the detection, generation, transmission and integration of signal information. Schwann cells, which form the myelin sheath, are separated by gaps of unsheathed axon over which the impulse travels as the signal propagates along the neuron. Action potentials propagate impulses along neurons. Membranes of neurons are polarized by the establishment of electrical potentials across the membranes. In response to a stimulus, Na+ and K+ gated channels sequentially open and cause the membrane to become locally depolarized. Na+/K+ pumps, powered by ATP, work to maintain membrane potential. A nerve impulse is an electrochemical event (initiated by various stimuli) that causes a change in neuron plasma membrane permeability, allowing sodium ions (Na+) to enter the cell (depolarization). Once begun, the action potential, or nerve impulse, continues over the entire surface of the cell. Electrical conditions of the resting state are restored by the diffusion of potassium ions (K+) out of the cell (repolarization). Ion concentrations of the resting state are restored by the sodium-potassium pump transmission of information between neurons occurs across synapses. Transmission across synapses involves chemical messengers called neurotransmitters. A neuron influences other neurons or effector cells by releasing neurotransmitters, chemicals that diffuse across the synaptic cleft and attach to membrane receptors on the postsynaptic cell. The result is opening of specific ion channels and activation or inhibition, depending on the neurotransmitter released and the target cell, causing a response.

Multiple Choice QuestionThe Steps Below refer to various stages in transmission at a chemical synapse:

1. Neurotransmitter binds with receptors associated with the postsynaptic membrane.

2. Calcium ions rush into neuron’s cytoplasm.

3. An action potential depolarizes the membrane of the axon terminal.

4. The ligand-gated ion channels open

5. The synaptic vesicles release neurotransmitter into synaptic cleft.

Which of the sequence of events is correct?

A. 1, 2, 3, 4, 5

B. 2, 3, 5, 4, 1

C. 3, 2, 5, 1, 4

D. 4, 3, 1, 2, 5

E. 5, 1, 2, 4, 3

Learning Log/ FRQ-style questionDescribe the basic pathway of information flow through neurons that causes you to turn your head when somebody calls your name.

Page 56: 7 th /8 th  Review Project Collection

LO: 3.47 Answer Key

Multiple Choice QuestionThe Steps Below refer to various stages in

transmission at a chemical synapse:1. Neurotransmitter binds with receptors

associated with the postsynaptic membrane.2. 2. Calcium ions rush into neuron’s cytoplasm.3. An action potential depolarizes the membrane of

the axon terminal.4. The ligand-gated ion channels open5. The synaptic vesicles release neurotransmitter into

synaptic cleft.• Which of the sequence of events is correct?A. 1, 2, 3, 4, 5B. 2, 3, 5, 4, 1C. 3, 2, 5, 1, 4D. 4, 3, 1, 2, 5E. 5, 1, 2, 4, 3

Learning Log/ FRQ-style questionDescribe the basic pathway of information flow through

neurons that causes you to turn your head when somebody calls your name. Draw a diagram for this process.

- Sensors in your ear transmit the external auditory stimuli to your brain. The stimuli is then carries the impulse from the ear to an interneuron. The activity of the interneuron process the information and enables you to recognize your name. In response, efferent neurons transmit signals which cause contraction of muscles in the neck that enable you to turn your head towards the stimuli.

Sample drawing

Page 57: 7 th /8 th  Review Project Collection

LO 3.38 The student is able to describe a model that expresses key elements to show how change in signal transduction can alter cellular response.

SP 1.5 The student can re-express key elements of natural phenomena across multiple representations in the domain.

Explanation: Visual models and representations are valuable resources for learning and investigating biological concepts and ideas. When given a specific biological process, the student is capable of creating correct representations and models using written explanations. The student must also be able to use models and representations to communicate information, make predictions, illustrate biological processes and concepts, and describe systems to exemplify complete understanding. For instance, the student must be able to present their knowledge via both written and visual explanations of key concepts of signal transduction and how transduction converts the signal to another form that can bring about a specific, desired response. This is done by a signal molecule binding to the receptor protein and changing it, then relay molecules aid in transduction , which triggers a specific cellular response. This information can then be used by the student to predict, with scientific evidence, how the response is altered. The student should be aware of the effects of a blocked or defective signal transduction. In order to do this, they must clearly illustrate the importance of relay proteins, including a scaffolding protein, protein kinases, and the receptor. Then, in either a written explanation or a clearly labeled drawing, show the effects of a defective or missing relay protein. This can be done with references to disorders such as WAS, which involves the absence of a relay protein.

MC Question: Relay proteins act as points of intersection or branching in signaling pathways. In the case of Wiskott- Aldrich syndrome (WAS), the absence of a relay protein can lead to symptoms such as bleeding or eczema. Studies have found that the WAS protein is located just beneath the cell surface. With your knowledge of relay proteins and signal transduction, which of the following theories best explains why a missing relay protein has such adverse affects. a.The relay protein acts as the signal’s means for transportation in a signaling pathway. Therefore, its absence disrupts the signaling pathways. Without the relay protein, the response cannot occur and causes immune cell behavior to not be controlled as easily. b.Without the relay protein, the protein kinases bind to proteins that activate symptoms of bleeding or eczema. c.The signaling pathway continues through signal amplification, but the response is negative instead of positive. d.The relay protein is absent near the cell surface because it is located in a different area of the cell. It aids in transduction of another signal that is set to respond with bleeding or eczema.

FRQ: Using the model shown to the right, a. Identify and describe the specific processes of cell

communication that are illustrated. b. Describe the effects of the removal of the relay molecules

in process number two. c. Describe the effects of a drug inhibiting the relay

molecules.

1 2 3

Alicia Paul

Page 58: 7 th /8 th  Review Project Collection

Answer Key LO 3.38MC Question: Relay proteins act as points of intersection or branching in signaling pathways. In the case of Wiskott Aldrich syndrome (WAS), the absence of a relay protein can lead to symptoms such as bleeding or eczema. Studies have found that the WAS protein is located just beneath the cell surface. With your knowledge of relay proteins and signal transduction, which of the following theories best explains why a missing relay protein has such adverse affects.

a.The relay protein acts as the signal’s means for transportation in a signaling pathway. Therefore, its absence disrupts the signaling pathways. Without the relay protein, the response cannot occur and causes immune cell behavior to not be controlled as easily.

b.Without the relay protein, the protein kinases bind to proteins that activate symptoms of bleeding or eczema.

c.The signaling pathway continues through signal amplification, but the response is negative instead of positive.

d.The relay protein is absent near the cell surface because it is located in a different area of the cell. It aids in transduction of another signal that is set to respond with bleeding or eczema. The symptoms of WAS likely arise mainly from the absence of the protein in immune system cells. The multifunctional relay protein acts as a point of intersection and a branch point in an intricate signal transduction network that controls the behavior of immune cells. Without the WAS protein, the cytoskeleton isn’t organized accurately and signaling pathways are disrupted, which leads to the symptoms.

FRQ: Using the model shown to the right,

a.Identify and describe the specific processes of cell communication that are illustrated.

b.Describe the effects of the removal of the relay molecules in process number two.

c.Describe the effects of a drug inhibiting the relay molecules.

a. The first process illustrated is reception. The target cell has detected the signal molecule from outside of the cell. The signal then binds to the receptor protein, changing it to initiate the second process, transduction. During transduction, the signal is converted to another form that brings about a specific cellular response. This is done by the use of relay proteins in a signal transduction pathway. The third process is the response. The signal has finally triggered a response, which is either cytoplasmic or nuclear.

b. Without relay molecules, they cannot complete their function as branching points or intersections in signaling pathways. They are crucial to the process of transduction because they alter the form of the signal molecule through a signal transduction pathway. Their absence would be prophylactic, causing the cellular response to not be triggered, as the signal would never change form.

c. The addition of a drug that inhibits relay molecules during signal transduction would also result in the cellular response to not be triggered. Inhibiting relay molecules inhibits transduction, which converts the signal molecule to a form that brings about the response. If this process does not occur, then the response cannot be made, leading to further problems in the body.

Alicia Paul

Page 59: 7 th /8 th  Review Project Collection

LO 4.2: The student is able to refine representations and models to explain how subcomponents of a biological polymer and their sequence determine properties of that polymer.

SP 1.3: The student can refine representations and models of natural or man-made phenomena and systems in the domain.

Explanation: The four biological polymers are carbohydrates, proteins, lipids and nucleic acids. The functional groups of these organic molecules are: OH, COOH, CO, NH₂, CH₃, SH and PO₄. Each of the four organic macromolecules have carbon back bones with the six functional groups attached that determine their function and shape. For example, polypeptides contain a carbon central molecule, with and amino group (NH₂), a carboxyl group, a hydrogen and a “R” group. The “R” group is a side chain that differs in each of the twenty amino acids. The “R” group ultimately decides the way the amino acid coils and folds around itself (tertiary structure) and other amino acids/molecules; which then decides the function of that amino acid/protein with two or more polypeptides (quaternary structure). (See diagrams to the right for further explanation)

M.C. Question: •Which subcomponent of DNA determines the direction in which DNA polymerase travels along a DNA strand?A: Phosphate GroupB: Deoxyribose SugarC: Nitrogenous BaseD: Lipid

Free Response Question: •Using words and diagrams, explain 3 structural differences between DNA and RNA.•Explain the functional differences between DNA and the three types of RNA. How does DNA and RNA’s structural differences help their functions?

Page 60: 7 th /8 th  Review Project Collection

Answer Key• Multiple Choice Question:• Which subcomponent of DNA determines

the direction in which DNA polymerase travels along a DNA strand?

A: Phosphate GroupB: Deoxyribose SugarC: Nitrogenous BaseD: Lipid

• Free Response Diagram:(sample/example)

• Free Response Question:• DNA is found solely in the nucleus of an animal cell. It is in

the shape of a double helix. DNA also has four bases: A, T, G and C, and it contains the sugar deoxyribose. RNA can be found in the nucleus and cytosol of an animal cell. It is single stranded and it’s four bases are A, U, G and C; it contains the sugar ribose.

• DNA’s function differs from RNA because DNA’s primary purpose is to store genetic information, while RNA’s primary function is to copy and deliver genetic information from the nucleus to the cytosol. TRNA is transfer RNA; it resides solely in the cytosol of the cell and translates codons to polypeptide chains. MRNA is messenger RNA; it carries information copied from the DNA in codons that will specify proteins to be made by the TRNA. RRNA is ribosomal RNA, which joins with proteins to form ribosomes that move along MRNA and start amino acid and protein chain assembly. RNA’s single stranded shape contributes to it’s ability to travel out of the nuclear envelope and contributes to it’s purpose of copying/delivering genetic information. The single stranded RNA is able to “attach” to the genetic material it’s copying and ‘’fit” between the ribosome easier during translation. DNA’s double stranded shape helps preserve genetic information from mutations or transcription mistakes.

Page 61: 7 th /8 th  Review Project Collection

LO 4.17 The student is able to analyze data to identify how molecular interactions affect structure and function. [See SP 5.1] SP 5.1 The student can analyze data to identify patterns or relationships.

Explanation: Interactions between molecules can significantly change their structure and function. For instance if a molecular system is changed in the least bit it can result in a change of the whole function of the system. For a chemical reaction to occur between two enzymes the substrate must fit in shape and charge to the enzyme’s active site. Other factors in enzyme function are cofactors and coenzymes which must be present and bind to the correct sites on the enzyme in order for it to become active. Molecules in the environment can also change the activity of an enzyme by binding irreversibly or reversibly to the active site. Data showing the concentration of a substrate or product can show whether or not there has been a change in the function of an enzyme.

MC Portion:Which of the following is a factor in an enzyme’s ability to catalyze a reaction:A) An enzyme will allow the reaction to go through a less stable transition state than normal.B) An enzyme will strengthen bonds in reactants.C) An enzyme provides a suitable environment and an appropriate reaction surface for the reaction to occur. D) When an enzyme binds reactants they can be positioned incorrectly, this can ultimately end in the

destruction of a substrate.

Free Response Style QuestionHow does the chemical structure of an enzyme

change and affect the catalytic activity of an enzyme? How does an enzyme catalyze a reaction? Use the figure given to help explain these two processes. Please include a description of what happens at each point included in the figure as well.

A. B. C. D.http://upload.wikimedia.org/wikipedia/commons/thumb/2/24/Induced_fit_diagram.svg/648px-Induced_fit_diagram.svg.png

Page 62: 7 th /8 th  Review Project Collection

LO 4.17 Answer Key

MC Portion:Which of the following is a factor in an enzyme’s ability

to catalyze a reaction:A) An enzyme will allow the reaction to go through a

less stable transition state than normal.B) An enzyme will strengthen bonds in reactants.C) An enzyme provides a suitable environment and an

appropriate reaction surface for the reaction to occur.

D) When an enzyme binds reactants they can be positioned incorrectly, this can ultimately end in the destruction of a substrate.

Free Response Style Question How does the chemical structure of an enzyme

change and affect the catalytic activity of an enzyme? How does an enzyme catalyze a reaction? Use the figure given to help explain these two processes. Please include a description of what happens at each point included in the figure provided.

At point A the substrate is entering the active site of the enzyme, at point B the enzyme changes shape slightly as it binds to the active site of the enzyme. At point C the enzymes are turned into products which are sent out of the active site of the enzyme at point D. The active site of the enzyme has a groove or pocket for the a special shape of a substrate, this is sometimes referred to as a lock and key. Modifiers of an enzyme such as cofactors, activators, and inhibitors can change the function of an enzyme. Cofactors are present at the active site to increase the rate of catalysis. An enzyme inhibitor is a molecule which binds to enzymes and decreases their activity. Enzyme Activators bind at the active site and increase their activity.

Page 63: 7 th /8 th  Review Project Collection

LO 4.15 The student is able to apply mathematical routines to quantities that describe interactions among living systems and their environment, which result in the movement of matter and energy. SP 2.2 The student can apply mathematical routines to quantities that describe natural phenomena.

Explanation: Organisms interact with their environment to gain the essentials for life. Energy in an ecosystem flows through and is degraded as it is passed through the food chain. Matter cycles in an ecosystem and is reused such as in photosynthesis and cellular respiration: the products of one reaction are the reactants of the other, and so the cycle of production and consumption continue. The climate of a region determine the organisms that live there, and therefore, the basis of the food chain. The amount of energy that can be made by the primary producers, and the the amount they use to maintain homeostasis, determines the amount of energy that can be passed on to the next trophic level and on through the food chain. Mathematical calculations can be applied to process the amount of energy and matter available at each level of consumption.

http://biology.tutorvista.com/ecology/food-web.htmlhttp://biology.tutorvista.com/ecology/food-web.html

M.C.Question: Given that the primary producer of a food chain initially takes in 500,000J of energy, and 35,000J is available to the primary consumer, what is the percent efficiency of the energy transfer between the two organisms?A) 10% C) 7%B) 3.5% D) 23%

Free Response: In the foothills of North Carolina, grasses makes up a significant proportion of the primary producers of the are. The grasses take in about 500,000J of energy and the efficiency of the energy transfer is approximately 4.5% throughout the food chain. A) Using the diagram to the right, create an energy pyramid that depicts the energy available at each tropic level.B)The efficiency of the energy transfer of the Snake decrease by 2.5%. What is the percent change of the amount of energy the snake can pass on to the hawk .

Page 64: 7 th /8 th  Review Project Collection

M.C.Question: Given that the primary producer of a food chain initially takes in 500,000J of energy, and 35,000J is available to the primary consumer, what is the percent efficiency of the energy transfer between the two organisms?A) 10% B) 3.5% C) 7% D) 23%

The amount of energy available to the next trophic level, divided by the amount of energy taken in, gives the proportion of energy that can be passed on. The proportion multiplied by 100 gives you the percent efficiency.

35,000J / 500,000J = 0.07 x 100 = 7% efficiency

Free Response: In the foothills of North Carolina, grasses makes up a significant proportion of the primary producers of the are. The grasses take in about 500,000J of energy and the efficiency of the energy transfer is approximately 4.5% throughout the food chain. A) Using the diagram to the right, create an energy pyramid that depicts the energy available at each tropic level.B)The efficiency of the energy transfer of the Snake decrease by 2.5%. What is the percent change of the amount of energy the snake can pass on to the hawk .

A)

500,000J500,000J

22,500J22,500J

1,012.5J1,012.5J

45.7J45.7J

GrassGrass

GrasshopperGrasshopperRabbitRabbitMouseMouse

LizardLizardSnakeSnake

HawkHawk

B) The percent efficiency decreases from 4.5% down to 2%; therefore, the amount of energy available to the next tropic level decreases from 45.7J to 20.3J. The percent change is the difference of the two energies divided by the original and multiplied by 100. The percent change of the amount of energy the snake can give to the next trophic level is 55.6% decrease.

4.5% - 2.5% = 2% 1,012.5J x .o2 = 20.3J

(45.7J - 20.25J) / 45.7J = 0.5557 x 100 = 55.6% decrease

SunSun

Page 65: 7 th /8 th  Review Project Collection

LO 4.15: The student is able to use visual representations to analyze situations or solve problems qualitatively to illustrate how interactions among living systems and with their environment result in the movement of matter and energy.

SP 1.4: The student can use representations and models to analyze situations or solve problems qualitatively and quantitatively.Explanation: Interactions among living systems and with their environment result in the movement of matter and energy. Within a living system, energy flows while matter is recycled. In an ecosystem, for example, many factors affect the organisms that live there. Climate change and different weather patterns can affect the success and productivity within an ecosystem, for both food webs and chains are dependent on primary productivity. Figures like the one to the right demonstrate the interdependence and flow of energy within a living system. Using the diagram can make it easier when makingpredictions about a system. Also, figures such as logistic models can be helpful with making predictions of the impact of change in biotic and abiotic factors. The curve may reflect population changes due to competition for resources, disease, or an accumulation of waste. Human activities also impact ecosystems. These harmful activities can decrease population sizes and contribute to habitat loss. Many adaptations of organisms are related to obtaining and using energy and matter in a particular environment.

Page 66: 7 th /8 th  Review Project Collection

Learning Log/FRQ-style Question: Use your knowledge of DDT and biomagnification to help answer the following questions. a)Propose one method as to how the DDT got into the water. b)Explain the short term effects of the DDT travelling through the food chain. c)Explain the long term effects of the DDT travelling through the food chain.

M.C. Question: How many kg of 2nd level carnivorous consumers can be supported by 100,000 kg of primary producers? A)100 B)1,000C)10D)1

Page 67: 7 th /8 th  Review Project Collection

Answer Key for LO 4.15

M.C. Question: How many kg of 2nd level carnivorous consumers can be supported by 100,000 kg of primary producers? A)100 B)1,000C)10D)1

This problem requires an understanding of the 10% rule with regards to energy and biomass flow within a living system. The number of kilograms of primary producers must be divided by 10 three times in order to calculate the amount of carnivorous consumers that can be supported by the autotrophs. This is because it is the third trophic level after the primary producers.

Learning Log/FRQ-style Question: Use your knowledge of DDT and biomagnification to help answer the following questions. a)Propose one method as to how the DDT got into the lake. b)Explain the short term effects of the DDT travelling through the food chain. c)Explain the long term effects of the DDT travelling through the food chain.

a) A farmer of a nearby field could have sprayed DDT on his plants to keep away pests. Frequent irrigation or a large storm could have produced runoff, leading the DDT filled water into the lake.

b) The DDT will begin to accumulate in zooplankton and fish. It will first appear in traces amounts but it will be magnified quickly. The fish-eating birds will be exposed to deadly amounts of DDT in their tissues.

c) The DDT levels will magnify until many of the fish die. As the fish populations decrease, the fish-eating birds will also decrease. The pond will experience a loss of many organisms as the DDT damages their tissues as well. The fish markets will suffer.

Page 68: 7 th /8 th  Review Project Collection

Page 1/3Learning ObjectiveLO 1.7 The student is able to justify data from mathematical models based on the Hardy-Weinberg equilibrium to analyze genetic drift and the effects of selection in the evolution of specific populations.Science Practice 2.1 The student can justify the selection of a mathematical routine to solve problems. ExplanationThe Hardy–Weinberg principle states that allele and genotype frequencies in a population will remain constant from generation to generation in the absence of other evolutionary influences. The Hardy-Weinberg equilibrium can be disrupted by a number of forces including mutations, natural selection, nonrandom mating, genetic drift, and gene flow. Genetic drift is one of the basic mechanisms for evolution. Genetic drift is the change in allele frequency in a population due to chance, over time an allele will become more or less common. Natural selection occurs when individuals with traits better suited to the environment reproduce more often, which allows their traits to be passed on. The Hardy- Weinberg equations are used to calculate the frequencies of the three possible genotypes in a population. Equations:p = the frequency of the dominant allele (represented by A)q = the frequency of the recessive allele (represented by a)For a population in genetic equilibrium:p + q = 1.0 (The sum of the frequencies of both alleles is 100%.)p 2 + 2pq + q2 = 1The three terms of this binomial expansion indicate the frequencies of the three genotypes:p2 = frequency of AA (homozygous dominant)2pq = frequency of Aa (heterozygous)q 2 = frequency of aa (homozygous recessive)Multiple ChoicePrior to a hurricane on an island there were 4000 mice; 3000 were homozygous dominant for brown fur, 500 were heterozygous and 500 were homozygous recessive. After the hurricane, the mouse population was 420; 210 are homozygous dominant for brown fur, 160 are heterozygous and 50 are homozygous recessive. What is the change in frequency of the dominant allele from the original population to the post hurricane population and what is this change in frequencies called?A) 0.16;crossing overB)0.70; crossing overC) 0.16; genetic driftD)0.005; genetic drift

Page 69: 7 th /8 th  Review Project Collection

Free Responsea) Describe Genetic Drift AND list the 5 criteria for Hardy Weinberg equilibrium and how they relate to genetic drift.b) Refer to the image and data chart below to answer the following questions.

Name and Explain the specific type of genetic drift that is being illustrated in the image and the effect it has on the overall population.

Now let’s say that the pink circle represents a dominant allele and the green circle represents the recessive allele, using the Hardy-Weinberg equations calculate the frequency of the homozygous dominant genotype, the heterozygous genotype, and the homozygous dominant genotype of both the original population and the new population.

Page 2/3

Chance event

Original Population New Population

Original Population New Population

Dominant allele frequency 0.50 0.80

Recessive allele frequency 0.50 0.20

Page 70: 7 th /8 th  Review Project Collection

Answers:Multiple Choice Answer:CFree Response Answer:1. a)Genetic Drift is the change of allele frequency in a population due to chance. The five conditions for Hardy- Weinberg Equilibrium are an extremely large population, no gene flow, no mutations, random mating, and non natural selection. A large population size relates to genetic drift because the smaller the population, the greater the probability that the allele frequencies will change due to chance and genetic drift will occur, so the larger a population the less likely that allele frequencies will change. Gene flow, which is the transfer of alleles between populations, can alter allele frequencies, if there is no gene flow there is no genetic drift. Mutations can modify the gene pool by introducing or removing genes from chromosomes or by changing one allele into another, so if no mutations occur the gene pool will not be altered and it will not contribute to genetic drift. Random mating does not occur because individuals preferentially choose mates with certain genotypes but if random mating did occur, certain traits would not be selected for and genetic drift would not occur. The last criteria, no natural selection, would inhibit genetic drift because this would mean that there were no differences in reproductive success of individuals carrying different genotypes so the allelic frequencies would not be altered. b) This image is showing the bottleneck effect, which is when a sudden change in the environment drastically reduces the size of a population and the affected population’s gene pool does not reflect the original population’s gene pool. The immediate effect of a population bottleneck is to decrease genetic diversity, promoting the effects of genetic drift. Original Population: homozygous dominant frequency= 0.25

heterozygous frequency=0.25 homozygous recessive frequency=0.5

New Population: homozygous dominant frequency= 0.64 heterozygous frequency=0.32 homozygous recessive frequency=0.04

Page 3/3

Page 71: 7 th /8 th  Review Project Collection

LO 2.3: The student is able to predict how changes in free energy availability affect organisms, populations and ecosystems.SP 6.4: The student can make claims and predictions about natural phenomena based on scientific theories and models.Explanation: Energy comes from the sun. Energy is made available in the trophic levels by producers or autotrophs. It is continuously lost because of metabolic activity and limits how much energy can be passed on to higher trophic levels. The Law of Thermodynamics explains that energy becomes more entropy as energy is transferred. About 10% of energy is passed on which is called the Rule of 10%. The energy is chemically bonded and converted into heat energy by plants. And allows heterotrophs to be able to receive energy.MC Question: If there were no autotrophs what would happen to the flow of energy involving NPP and GPP in the trophic levels?A. The NPP and GPP could still exist.B. The NPP would be created by the heterotrophs and the GPP would be left over for the next trophic levels. C. The GPP would be created by the heterotrophs and the NPP would be left over for the next trophic levels.D. The NPP would not existed and there would be no GPP for the next trophic levels.

Learning Log/ FRQ- style Question: Explain how producers and non photosythenthic organisms obtain energy. Explain why there has to be so a large amount of lower trophic level species in order to feed the small amount of higher trophic levels. How much GPP would be left for the secondary consumer if the producers consumed 200,000 J of energy? Tertiary consumer?

Page 72: 7 th /8 th  Review Project Collection

ANSWER KEY- LO 4.1MC Question: If there were no autotrophs what would happen to the flow of energy involving NPP and GPP in the trophic levels?A. The NPP and GPP could still exist.B. The NPP would be created by the heterotrophs and the GPP would be left over for the next trophic levels. C . The GPP would be created by the heterotrophs and the NPP would be left over for the next trophic levels.D. The NPP would not existed and there would be no GPP for the next trophic levels.

- Why? Producers convert energy (NPP) from the sun and allow the consumers to access the leftover energy (GPP) after the producers take in energy.

Explain how producers and non photosynthetic organisms obtain energy. Explain why there has to be so a large amount of lower trophic level species in order to feed the small amount of higher trophic levels. How much energy would be left for the secondary consumer if the producers consumed 200,000 J of energy? Tertiary consumer? - Producer take in the energy from the sun which is the source of the energy pyramid and allows the energy to be able to be used by doing photosynthesis. Non photosynthetic organisms such as bacteria that live deep where there is no sun light use a process called chemosynthesis, which the synthesis of carbohydrate from carbon dioxide and water using energy obtained from the chemical oxidation of simple inorganic compounds. Energy is lost mainly through heat, so not much is transferred to the next trophic level, only about 10% is transferred. The producer have to convert through aerobic respiration, C6H12O6 + 6 O2 6 CO2 + 6 H2O, in order for the higher trophic levels to access it. Since the energy being transferred isn’t much there has to be many producers and lower trophic levels.

Page 73: 7 th /8 th  Review Project Collection

200,000 J x .01 (10%) = 20,000 J200,000 J - 20,000 J = 180,000 J for the secondary consumers180,000 J x .1 (10%)= 18,000 J180,000 J – 18,000 J =162,000 J for the tertiary consumersThere would be 180,000 J of energy left for the secondary consumers and 162,000 J of energy for the tertiary consumers.

Page 74: 7 th /8 th  Review Project Collection

LO 2.2: The student is able to justify a scientific claim that free energy is required for living systems to maintain organization, to grow or to reproduce, but that multiple strategies exist in different living systems.

SP 6.1: The student can justify claims with evidence.

Explanation: All living systems require constant input of free energy. The order is maintained when free energy is constantly available, any loss of order or free energy will result in death. Living Systems do not violate the 2nd law of thermodynamics: Energy transfer or transformation increases disorder (entropy) of the universe. Organisms use free energy to keep organization, grow and also reproduce. Also, reproducing and raising offspring require free energy beyond what is required for maintenance and growth; such variance can affect populations and distribution among the ecosystem.

(a) First law of thermodynamics: Energy can be transferred or transformed but neither created nor destroyed. For example, the chemical (potential) energy in food will be converted to the kinetic energy of the cheetah’s movement in (b).

Second law of thermodynamics: Every energy transfer or transformation increasesthe disorder (entropy) of the universe. For example, disorder is added to the cheetah’ssurroundings in the form of heat and the small molecules that are the by-productsof metabolism.

(b)

Chemicalenergy

Heat co2

H2O+

Page 75: 7 th /8 th  Review Project Collection

FRQ: Thermodynamic is the study of energy transformations that occur in a collection of matter. Discuss the first and second laws of Thermodynamics, include in your answer how energy can help lead to growth.

Multiple Choice Question: 1) In What situation would a diver have the most potential energy?

a. Standing on a platform diving into the water.

b. In the process of diving into the water, diver is in the air.

c. Swimming in the water

d. Climbing up steps to dive again.

On the platform, a diverhas more potential energy.

Diving converts potentialenergy to kinetic energy.

Climbing up converts kineticenergy of muscle movement to potential energy.

In the water, a diver has less potential energy.

Page 76: 7 th /8 th  Review Project Collection

Multiple Choice Question:

1) In What situation would a diver have the most potential energy?

a. Standing on a platform diving into the water.

b. In the process of diving into the water, diver is in the air.

c. Swimming in the water

d. Climbing up steps to dive again.